Time Left - 20:00 mins

Critical Reasoning Quiz II CLAT 2022 II 24.09.2021

Attempt now to get your rank among 227 students!

Question 1

A statement is followed by two assumptions numbered I and II. An assumption is something supposed. You have to consider the statement and the following assumptions and decide which of the assumptions is implicit in the statement.
Statement - ‘Music Loving Club’ has announced a wide variety of programme for the visiting musicians.
Assumptions -
I. A large number of people are interested to participate in the programme
II. The artists may attract a large number of spectators.

Question 2

Direction: Read the excerpt carefully and answer the questions that follows:
The earth’s resources are being depleted much too fast. To correct this, the United States must reduce its resource consumption at present levels for many years to come.
Which of the following, if true, would most strengthen the argument above?

Question 3

Data on planes returning from bombing missions was used to study of the vulnerability of airplanes to enemy fire. Analyzing the pattern and frequency of hits from enemy gunfire, it was seen that some parts of planes were hit disproportionately more often than other parts. How could these planes be optimally reinforced with armour plating? There were tradeoffs to consider. Every addition of plating added to the weight of the plane, decreasing its performance. Therefore, reinforcements needed to be added only to the most vulnerable areas of the planes.

Which of the following can be concluded from the above?

Question 4

Direction: In the question below is given a statement followed by some assumptions. An assumption is something supposed or taken for granted. You have to consider the statement and the following assumptions and decide which of the assumptions is implicit in the statement.
Statement:
India had a long and honorable history of writers on social concerns but the problem is that Indians don’t have a sense of history. 

Assumptions:
I. The individual bias of an author affects the spirit of history but this is not the case with social issues.
II. The present issues of social concern cannot constitute a part of history.

Question 5

The cases of food poisoning due to the consumption of liquor in rural areas are far greater than such cases in urban areas. Hence it can be concluded that the liquor supplied in rural areas is of a low quality than that supplied in urban areas. 

Which of the following statements if true will most weaken the given argument? 

I. The prices of liquor are lower in rural areas than in urban areas.
II. The percentage of people consuming liquor is more in rural areas than in urban areas.
III. There are many unauthorized spurious liquor shops in rural areas.
IV. The number of people suffering from food poisoning due to consumption of liquor as a percentage of people who consume liquor is almost equal for both urban areas and rural areas.

Question 6

The cases of food poisoning due to the consumption of liquor in rural areas are far greater than such cases in urban areas. Hence it can be concluded that the liquor supplied in rural areas is of a low quality than that supplied in urban areas.

Which of the following statements numbered if true will strengthen the given argument? 

I. The prices of liquor are lower in rural areas than in urban areas.
II. Percentage of people consuming liquor is more in rural areas than in urban areas.
III. There are many unauthorized spurious liquor shops in rural areas.
IV. Number of people suffering from food poisoning due to consumption of liquor as a percentage of people who consume liquor is almost equal for both urban areas and rural areas.

Question 7

Statement: The district administration has issued a circular to all the farmers under its jurisdiction advising them not to use pesticides indiscriminately as it may pollute the groundwater.

Which of the following conclusions can be assumed from the given statement?

I. People may stop using groundwater if the farmers continue to use pesticides indiscriminately.
II. The polluted groundwater can cause health problems.
III. Farmers may refrain from using pesticides indiscriminately.
IV. People in the district are majorly dependent on groundwater for drinking use.

Question 8

Direction: These questions are based on the statement given below and the sentences labelled (A), (B), (C) and (D) as given below.

The number of people diagnosed with dengue fever (which is contracted from the bite of an infected mosquito) in North India this year is twice the number diagnosed last year. The authorities have concluded that measures to control the mosquito population have failed in this region.
A) An effective diagnostic test was introduced about nine months ago.
B) The incidence (number of cases per thousand) of malaria, also contracted from mosquito bites, has increased.
C) A very high proportion of the cases were diagnosed in people of neighboring country.
D) More cases are now reported because of increases in administrative efficiency.
Which of the above statements numbered (A), (B), (C) and (D) if true will weaken the given argument?

Question 9

Direction: These questions are based on the statement given below and the sentences labelled (A), (B), (C) and (D) as given below.

The number of people diagnosed with dengue fever (which is contracted from the bite of an infected mosquito) in North India this year is twice the number diagnosed last year. The authorities have concluded that measures to control the mosquito population have failed in this region.
A) An effective diagnostic test was introduced about nine months ago.
B) The incidence (number of cases per thousand) of malaria, also contracted from mosquito bites, has increased.
C) A very high proportion of the cases were diagnosed in people of neighboring country.
D) More cases are now reported because of increases in administrative efficiency.
Which of the above statements numbered (B), (C) and (D) if true will most weaken the given argument?

Question 10

Direction: In the question below, a statement is given followed by two arguments. Choose the most appropriate option depending on which argument strengthens the given statement.
Statement: 
Should the government provide books to all the students up to the Xth standard at zero cost in the schools across India?

Arguments:
I. No, as people who can afford the books shall be unduly benefitted by this move.
II. Yes, this will considerably improve the level of education and also will reduce school dropout rates.
  • 227 attempts
  • 2 upvotes
  • 1 comment
Sep 24CLAT UG